Angular speed of rod shot by bullet

In summary, the conversation discusses the application of conservation of linear momentum and circular motion formulas in solving a problem involving a bullet hitting a rod. The experts also mention the importance of considering energy conservation and the overspecification of the problem. Ultimately, it is concluded that angular momentum conservation is the most relevant approach for solving the problem.
  • #1
songoku
2,294
325
Homework Statement
A rod with mass ##M## and length ##L## meter is pivoted at its center and put in vertical position, then a bullet of mass ##m## with initial speed ##u## is directed at a point ##x## meter from the top of the rod (##x## < ##0.5~L##) and the bullet will penetrates through the rod with final speed ##v##. Considering no energy lost and ignoring mass of rod lost when the bullet penetrates, find the angular speed of the rod
Relevant Equations
Conservation of linear momentum

Conservation of angular momentum

Circular motion
1) Applying conservation of linear momentum:

$$m.u = M.V + m.v$$

where ##V## is final linear speed of the rod

$$V=\frac{m.u-m.v}{M}$$2) Applying formula of circular motion:
$$V=\omega . r$$
$$\omega = \frac{\left(\frac{mu-mv}{M} \right)}{\frac{1}{2}L-x}$$

Is this correct?And can this be solved by using conservation of angular momentum? This is what I tried:

Applying conservation of angular momentum:
$$m.u.(\frac{1}{2}L-x)=m.v.(\frac{1}{2}L-x)+I \omega$$
$$(\frac{1}{2}L-x)(m.u-m.v)=\frac{1}{12} ML^2 \omega$$
$$\omega = \frac{(\frac{1}{2}L-x)(m.u-m.v)}{\frac{1}{12} ML^2}$$I am really not sure about my working. Thanks
 
Last edited:
Physics news on Phys.org
  • #2
Linear momentum is not conserved because the pivot exerts an external force. Angular momentum about the pivot is conserved assuming that the bullet goes through very fast.
 
Last edited:
  • Like
Likes songoku and Delta2
  • #3
Something tells me you have to use conservation of energy because the statements says no energy lost.
 
  • Like
Likes songoku
  • #4
Angular momentum of the whole system is conserved. Only that is needed.
Condition x≤L/2 is strange; condition about the energy is strange as well
$$xmu=J\omega+xmv$$
 
  • Like
Likes songoku and kuruman
  • #5
Delta2 said:
Something tells me you have to use conservation of energy because the statements says no energy lost.
I agree that that's what the problem says but the issue is that if we conserve energy, the answer is different from what one gets from angular momentum conservation. Angular momentum is conserved because the torque due to gravity about the pivot can be ignored if the bullet goes through fast enough.

The energy conservation route gives a result that is independent of the distance ##x## from the pivot where the bullet strikes. So why is that distance given? Also, unless the energy transfer from bullet to rod is mediated by conservative forces, I don't see how energy can be conserved. Bullets going through rods usually do not exert conservative forces.

I favor ignoring the energy conservation clause and proceeding with angular momentum conservation.
 
  • Like
Likes songoku
  • #6
wrobel said:
Angular momentum of the whole system is conserved. Only that is needed.
Condition x≤L/2 is strange; condition about the energy is strange as well
$$xmu=J\omega+xmv$$
Shouldn't the distance measured from the pivot, so ##\frac{1}{2}L - x## since ##x## is measured from top of rod?

Thanks
 
  • #7
kuruman said:
I agree that that's what the problem says but the issue is that if we conserve energy, the answer is different from what one gets from angular momentum conservation. Angular momentum is conserved because the torque due to gravity about the pivot can be ignored if the bullet goes through fast enough.

The energy conservation route gives a result that is independent of the distance ##x## from the pivot where the bullet strikes. So why is that distance given? Also, unless the energy transfer from bullet to rod is mediated by conservative forces, I don't see how energy can be conserved. Bullets going through rods usually do not exert conservative forces.

I favor ignoring the energy conservation clause and proceeding with angular momentum conservation.
Yes, the problem is overspecified. We are given u, v, m, M, x and L. From these we can calculate the energy lost.
Conversely, we can take energy as conserved and calculate v.
 
  • Like
Likes songoku
  • #8
songoku said:
Shouldn't the distance measured from the pivot, so ##\frac{1}{2}L - x## since ##x## is measured from top of rod?

Thanks
Yes, it should.
 
  • Like
Likes songoku
  • #9
haruspex said:
Yes, the problem is overspecified. We are given u, v, m, M, x and L. From these we can calculate the energy lost.
Conversely, we can take energy as conserved and calculate v.
Will the energy lost be: ##\frac1 2 mu^2 - \frac 1 2 mv^2 - \frac 1 2 J \omega^2## where ##J## is the moment of inertia of the rod?

Thanks
 
  • #10
songoku said:
Will the energy lost be: ##\frac1 2 mu^2 - \frac 1 2 mv^2 - \frac 1 2 J \omega^2## where ##J## is the moment of inertia of the rod?

Thanks
It would be that with ##\omega## being what you get using angular momentum conservation.
 
  • Like
Likes songoku
  • #11
I wrote the formula for the following configuration

cvbc.png
 
  • Like
Likes songoku
  • #12
Thank very much for all the help and explanation kuruman, Delta2, wrobel, haruspex
 

1. What is the definition of angular speed?

Angular speed is the rate at which an object rotates or moves around a fixed point, measured in radians per second.

2. How is the angular speed of a rod shot by a bullet calculated?

The angular speed of a rod shot by a bullet can be calculated by dividing the linear speed of the bullet by the distance from the fixed point to the center of mass of the rod.

3. Does the mass of the bullet affect the angular speed of the rod?

Yes, the mass of the bullet does affect the angular speed of the rod. A heavier bullet will have a greater linear speed, resulting in a higher angular speed of the rod.

4. What other factors can affect the angular speed of the rod shot by a bullet?

Other factors that can affect the angular speed of the rod include the length and shape of the rod, the distance between the fixed point and the center of mass of the rod, and the angle at which the bullet strikes the rod.

5. How is the angular speed of the rod related to the bullet's impact on the rod?

The angular speed of the rod is directly related to the bullet's impact on the rod. A higher angular speed means the rod will rotate faster, resulting in a more forceful impact from the bullet.

Similar threads

  • Introductory Physics Homework Help
10
Replies
335
Views
8K
  • Introductory Physics Homework Help
Replies
3
Views
223
  • Introductory Physics Homework Help
Replies
3
Views
1K
Replies
13
Views
899
  • Introductory Physics Homework Help
Replies
13
Views
1K
  • Introductory Physics Homework Help
Replies
10
Views
906
  • Introductory Physics Homework Help
Replies
6
Views
694
  • Introductory Physics Homework Help
2
Replies
54
Views
3K
  • Introductory Physics Homework Help
Replies
14
Views
1K
  • Introductory Physics Homework Help
Replies
10
Views
3K
Back
Top